Show that two primitives/ antiderivatives are related via a constant












0












$begingroup$



Let $I subset R$ be an interval. A differentiable function $F: I rightarrow R$ is called a
primitive for the function $f : I rightarrow mathbb{R}$ if
$$F'
(x) = f(x)$$
for all $x in I$.



Show: If $F_ 1$ and $F_2$ are two primitives for $f$ on $I$ then there is a constant
$C in mathbb{R}$ such that $F_2 equiv F_1 + C$, i.e. $F_2(x) = F_1(x) + C$ for all $x in I$.




What we have covered so far is the formal definition of the derivative in terms of the limit:
$$ lim_{h rightarrow 0}frac{F_1(x+h)-F_1(x)}{h}= f(x)$$
$$ lim_{h rightarrow 0}frac{F_2(x+h)-F_2(x)}{h}= f(x)$$



I do not see how I can prove firsthand that these two functions differ by a constant. I know it to be true from my pre-calculus and calculus experience, but how would one make the argument from an analysis point of view, can someone give me a hint as to where this constant 'appears'.





We normally just define $F_2=F_1 +C$ and then it would follow immediately that these two functions have the same derivative.










share|cite|improve this question









$endgroup$












  • $begingroup$
    math.stackexchange.com/questions/1862231/…
    $endgroup$
    – Hans Lundmark
    Dec 11 '18 at 10:44






  • 1




    $begingroup$
    It is essential that the domain is an intervall.
    $endgroup$
    – gammatester
    Dec 11 '18 at 10:45










  • $begingroup$
    AH, the glorious $MVT$, cool!
    $endgroup$
    – Wesley Strik
    Dec 11 '18 at 10:46
















0












$begingroup$



Let $I subset R$ be an interval. A differentiable function $F: I rightarrow R$ is called a
primitive for the function $f : I rightarrow mathbb{R}$ if
$$F'
(x) = f(x)$$
for all $x in I$.



Show: If $F_ 1$ and $F_2$ are two primitives for $f$ on $I$ then there is a constant
$C in mathbb{R}$ such that $F_2 equiv F_1 + C$, i.e. $F_2(x) = F_1(x) + C$ for all $x in I$.




What we have covered so far is the formal definition of the derivative in terms of the limit:
$$ lim_{h rightarrow 0}frac{F_1(x+h)-F_1(x)}{h}= f(x)$$
$$ lim_{h rightarrow 0}frac{F_2(x+h)-F_2(x)}{h}= f(x)$$



I do not see how I can prove firsthand that these two functions differ by a constant. I know it to be true from my pre-calculus and calculus experience, but how would one make the argument from an analysis point of view, can someone give me a hint as to where this constant 'appears'.





We normally just define $F_2=F_1 +C$ and then it would follow immediately that these two functions have the same derivative.










share|cite|improve this question









$endgroup$












  • $begingroup$
    math.stackexchange.com/questions/1862231/…
    $endgroup$
    – Hans Lundmark
    Dec 11 '18 at 10:44






  • 1




    $begingroup$
    It is essential that the domain is an intervall.
    $endgroup$
    – gammatester
    Dec 11 '18 at 10:45










  • $begingroup$
    AH, the glorious $MVT$, cool!
    $endgroup$
    – Wesley Strik
    Dec 11 '18 at 10:46














0












0








0





$begingroup$



Let $I subset R$ be an interval. A differentiable function $F: I rightarrow R$ is called a
primitive for the function $f : I rightarrow mathbb{R}$ if
$$F'
(x) = f(x)$$
for all $x in I$.



Show: If $F_ 1$ and $F_2$ are two primitives for $f$ on $I$ then there is a constant
$C in mathbb{R}$ such that $F_2 equiv F_1 + C$, i.e. $F_2(x) = F_1(x) + C$ for all $x in I$.




What we have covered so far is the formal definition of the derivative in terms of the limit:
$$ lim_{h rightarrow 0}frac{F_1(x+h)-F_1(x)}{h}= f(x)$$
$$ lim_{h rightarrow 0}frac{F_2(x+h)-F_2(x)}{h}= f(x)$$



I do not see how I can prove firsthand that these two functions differ by a constant. I know it to be true from my pre-calculus and calculus experience, but how would one make the argument from an analysis point of view, can someone give me a hint as to where this constant 'appears'.





We normally just define $F_2=F_1 +C$ and then it would follow immediately that these two functions have the same derivative.










share|cite|improve this question









$endgroup$





Let $I subset R$ be an interval. A differentiable function $F: I rightarrow R$ is called a
primitive for the function $f : I rightarrow mathbb{R}$ if
$$F'
(x) = f(x)$$
for all $x in I$.



Show: If $F_ 1$ and $F_2$ are two primitives for $f$ on $I$ then there is a constant
$C in mathbb{R}$ such that $F_2 equiv F_1 + C$, i.e. $F_2(x) = F_1(x) + C$ for all $x in I$.




What we have covered so far is the formal definition of the derivative in terms of the limit:
$$ lim_{h rightarrow 0}frac{F_1(x+h)-F_1(x)}{h}= f(x)$$
$$ lim_{h rightarrow 0}frac{F_2(x+h)-F_2(x)}{h}= f(x)$$



I do not see how I can prove firsthand that these two functions differ by a constant. I know it to be true from my pre-calculus and calculus experience, but how would one make the argument from an analysis point of view, can someone give me a hint as to where this constant 'appears'.





We normally just define $F_2=F_1 +C$ and then it would follow immediately that these two functions have the same derivative.







real-analysis






share|cite|improve this question













share|cite|improve this question











share|cite|improve this question




share|cite|improve this question










asked Dec 11 '18 at 10:37









Wesley StrikWesley Strik

2,057423




2,057423












  • $begingroup$
    math.stackexchange.com/questions/1862231/…
    $endgroup$
    – Hans Lundmark
    Dec 11 '18 at 10:44






  • 1




    $begingroup$
    It is essential that the domain is an intervall.
    $endgroup$
    – gammatester
    Dec 11 '18 at 10:45










  • $begingroup$
    AH, the glorious $MVT$, cool!
    $endgroup$
    – Wesley Strik
    Dec 11 '18 at 10:46


















  • $begingroup$
    math.stackexchange.com/questions/1862231/…
    $endgroup$
    – Hans Lundmark
    Dec 11 '18 at 10:44






  • 1




    $begingroup$
    It is essential that the domain is an intervall.
    $endgroup$
    – gammatester
    Dec 11 '18 at 10:45










  • $begingroup$
    AH, the glorious $MVT$, cool!
    $endgroup$
    – Wesley Strik
    Dec 11 '18 at 10:46
















$begingroup$
math.stackexchange.com/questions/1862231/…
$endgroup$
– Hans Lundmark
Dec 11 '18 at 10:44




$begingroup$
math.stackexchange.com/questions/1862231/…
$endgroup$
– Hans Lundmark
Dec 11 '18 at 10:44




1




1




$begingroup$
It is essential that the domain is an intervall.
$endgroup$
– gammatester
Dec 11 '18 at 10:45




$begingroup$
It is essential that the domain is an intervall.
$endgroup$
– gammatester
Dec 11 '18 at 10:45












$begingroup$
AH, the glorious $MVT$, cool!
$endgroup$
– Wesley Strik
Dec 11 '18 at 10:46




$begingroup$
AH, the glorious $MVT$, cool!
$endgroup$
– Wesley Strik
Dec 11 '18 at 10:46










3 Answers
3






active

oldest

votes


















2












$begingroup$

Hint: What is the derivative of $F_1-F_2$?






share|cite|improve this answer









$endgroup$













  • $begingroup$
    Zero of course, so we can define a new function $H(x)=F_2 -F_1$ such that $H'(x) =0$
    $endgroup$
    – Wesley Strik
    Dec 11 '18 at 10:49












  • $begingroup$
    And then apply the MVT to the interval.
    $endgroup$
    – Wesley Strik
    Dec 11 '18 at 10:49






  • 1




    $begingroup$
    @WesleyStrik I think you're thinking too complicated. If $H'(x)=0$, then $H(x)=C$ for some constant $C$. What does this say about the relationship between $F_1$ and $F_2$?
    $endgroup$
    – Arthur
    Dec 11 '18 at 10:52












  • $begingroup$
    It means the function $F_1 - F_2$ is constant, see my proof below using the MVT.
    $endgroup$
    – Wesley Strik
    Dec 11 '18 at 12:20





















1












$begingroup$

Hint:



If $F_1,F_2$ are primitives of $f$, by linearity of differentiation,



$$(F_1(x)-F_2(x))'=F'_1(x)-F'_2(x)=f(x)-f(x)=0.$$



Remains to prove that the antiderivative of $0$ is a constant function.






share|cite|improve this answer









$endgroup$





















    0












    $begingroup$

    Let $G(x)= F_1 - F_2.$



    We observe that $G'(x)=0$. We claim this means that $G(x)$ is constant.



    Suppose on the contrary the function is non-constant this means there must exist $a, b in I$ and where $a<b$ such that $G(a) neq G(b)$.



    We consider $[a,b]$. Now We apply the mean value theorem to this interval and the function $G(x)$. We get that there must exist a $c$ such that:
    Which is a contradiction.
    $$G'(c)= frac{G(b)-G(a)}{b-a} land G'(c)=0,$$
    $$ frac{G(b)-G(a)}{b-a}=0 implies G(a)=G(b). $$
    We conclude that $G(x)$ is constant. Thus, $F_2$ and $F_1$ differ by a constant $square$.






    share|cite|improve this answer











    $endgroup$













      Your Answer





      StackExchange.ifUsing("editor", function () {
      return StackExchange.using("mathjaxEditing", function () {
      StackExchange.MarkdownEditor.creationCallbacks.add(function (editor, postfix) {
      StackExchange.mathjaxEditing.prepareWmdForMathJax(editor, postfix, [["$", "$"], ["\\(","\\)"]]);
      });
      });
      }, "mathjax-editing");

      StackExchange.ready(function() {
      var channelOptions = {
      tags: "".split(" "),
      id: "69"
      };
      initTagRenderer("".split(" "), "".split(" "), channelOptions);

      StackExchange.using("externalEditor", function() {
      // Have to fire editor after snippets, if snippets enabled
      if (StackExchange.settings.snippets.snippetsEnabled) {
      StackExchange.using("snippets", function() {
      createEditor();
      });
      }
      else {
      createEditor();
      }
      });

      function createEditor() {
      StackExchange.prepareEditor({
      heartbeatType: 'answer',
      autoActivateHeartbeat: false,
      convertImagesToLinks: true,
      noModals: true,
      showLowRepImageUploadWarning: true,
      reputationToPostImages: 10,
      bindNavPrevention: true,
      postfix: "",
      imageUploader: {
      brandingHtml: "Powered by u003ca class="icon-imgur-white" href="https://imgur.com/"u003eu003c/au003e",
      contentPolicyHtml: "User contributions licensed under u003ca href="https://creativecommons.org/licenses/by-sa/3.0/"u003ecc by-sa 3.0 with attribution requiredu003c/au003e u003ca href="https://stackoverflow.com/legal/content-policy"u003e(content policy)u003c/au003e",
      allowUrls: true
      },
      noCode: true, onDemand: true,
      discardSelector: ".discard-answer"
      ,immediatelyShowMarkdownHelp:true
      });


      }
      });














      draft saved

      draft discarded


















      StackExchange.ready(
      function () {
      StackExchange.openid.initPostLogin('.new-post-login', 'https%3a%2f%2fmath.stackexchange.com%2fquestions%2f3035158%2fshow-that-two-primitives-antiderivatives-are-related-via-a-constant%23new-answer', 'question_page');
      }
      );

      Post as a guest















      Required, but never shown

























      3 Answers
      3






      active

      oldest

      votes








      3 Answers
      3






      active

      oldest

      votes









      active

      oldest

      votes






      active

      oldest

      votes









      2












      $begingroup$

      Hint: What is the derivative of $F_1-F_2$?






      share|cite|improve this answer









      $endgroup$













      • $begingroup$
        Zero of course, so we can define a new function $H(x)=F_2 -F_1$ such that $H'(x) =0$
        $endgroup$
        – Wesley Strik
        Dec 11 '18 at 10:49












      • $begingroup$
        And then apply the MVT to the interval.
        $endgroup$
        – Wesley Strik
        Dec 11 '18 at 10:49






      • 1




        $begingroup$
        @WesleyStrik I think you're thinking too complicated. If $H'(x)=0$, then $H(x)=C$ for some constant $C$. What does this say about the relationship between $F_1$ and $F_2$?
        $endgroup$
        – Arthur
        Dec 11 '18 at 10:52












      • $begingroup$
        It means the function $F_1 - F_2$ is constant, see my proof below using the MVT.
        $endgroup$
        – Wesley Strik
        Dec 11 '18 at 12:20


















      2












      $begingroup$

      Hint: What is the derivative of $F_1-F_2$?






      share|cite|improve this answer









      $endgroup$













      • $begingroup$
        Zero of course, so we can define a new function $H(x)=F_2 -F_1$ such that $H'(x) =0$
        $endgroup$
        – Wesley Strik
        Dec 11 '18 at 10:49












      • $begingroup$
        And then apply the MVT to the interval.
        $endgroup$
        – Wesley Strik
        Dec 11 '18 at 10:49






      • 1




        $begingroup$
        @WesleyStrik I think you're thinking too complicated. If $H'(x)=0$, then $H(x)=C$ for some constant $C$. What does this say about the relationship between $F_1$ and $F_2$?
        $endgroup$
        – Arthur
        Dec 11 '18 at 10:52












      • $begingroup$
        It means the function $F_1 - F_2$ is constant, see my proof below using the MVT.
        $endgroup$
        – Wesley Strik
        Dec 11 '18 at 12:20
















      2












      2








      2





      $begingroup$

      Hint: What is the derivative of $F_1-F_2$?






      share|cite|improve this answer









      $endgroup$



      Hint: What is the derivative of $F_1-F_2$?







      share|cite|improve this answer












      share|cite|improve this answer



      share|cite|improve this answer










      answered Dec 11 '18 at 10:41









      ArthurArthur

      116k7116198




      116k7116198












      • $begingroup$
        Zero of course, so we can define a new function $H(x)=F_2 -F_1$ such that $H'(x) =0$
        $endgroup$
        – Wesley Strik
        Dec 11 '18 at 10:49












      • $begingroup$
        And then apply the MVT to the interval.
        $endgroup$
        – Wesley Strik
        Dec 11 '18 at 10:49






      • 1




        $begingroup$
        @WesleyStrik I think you're thinking too complicated. If $H'(x)=0$, then $H(x)=C$ for some constant $C$. What does this say about the relationship between $F_1$ and $F_2$?
        $endgroup$
        – Arthur
        Dec 11 '18 at 10:52












      • $begingroup$
        It means the function $F_1 - F_2$ is constant, see my proof below using the MVT.
        $endgroup$
        – Wesley Strik
        Dec 11 '18 at 12:20




















      • $begingroup$
        Zero of course, so we can define a new function $H(x)=F_2 -F_1$ such that $H'(x) =0$
        $endgroup$
        – Wesley Strik
        Dec 11 '18 at 10:49












      • $begingroup$
        And then apply the MVT to the interval.
        $endgroup$
        – Wesley Strik
        Dec 11 '18 at 10:49






      • 1




        $begingroup$
        @WesleyStrik I think you're thinking too complicated. If $H'(x)=0$, then $H(x)=C$ for some constant $C$. What does this say about the relationship between $F_1$ and $F_2$?
        $endgroup$
        – Arthur
        Dec 11 '18 at 10:52












      • $begingroup$
        It means the function $F_1 - F_2$ is constant, see my proof below using the MVT.
        $endgroup$
        – Wesley Strik
        Dec 11 '18 at 12:20


















      $begingroup$
      Zero of course, so we can define a new function $H(x)=F_2 -F_1$ such that $H'(x) =0$
      $endgroup$
      – Wesley Strik
      Dec 11 '18 at 10:49






      $begingroup$
      Zero of course, so we can define a new function $H(x)=F_2 -F_1$ such that $H'(x) =0$
      $endgroup$
      – Wesley Strik
      Dec 11 '18 at 10:49














      $begingroup$
      And then apply the MVT to the interval.
      $endgroup$
      – Wesley Strik
      Dec 11 '18 at 10:49




      $begingroup$
      And then apply the MVT to the interval.
      $endgroup$
      – Wesley Strik
      Dec 11 '18 at 10:49




      1




      1




      $begingroup$
      @WesleyStrik I think you're thinking too complicated. If $H'(x)=0$, then $H(x)=C$ for some constant $C$. What does this say about the relationship between $F_1$ and $F_2$?
      $endgroup$
      – Arthur
      Dec 11 '18 at 10:52






      $begingroup$
      @WesleyStrik I think you're thinking too complicated. If $H'(x)=0$, then $H(x)=C$ for some constant $C$. What does this say about the relationship between $F_1$ and $F_2$?
      $endgroup$
      – Arthur
      Dec 11 '18 at 10:52














      $begingroup$
      It means the function $F_1 - F_2$ is constant, see my proof below using the MVT.
      $endgroup$
      – Wesley Strik
      Dec 11 '18 at 12:20






      $begingroup$
      It means the function $F_1 - F_2$ is constant, see my proof below using the MVT.
      $endgroup$
      – Wesley Strik
      Dec 11 '18 at 12:20













      1












      $begingroup$

      Hint:



      If $F_1,F_2$ are primitives of $f$, by linearity of differentiation,



      $$(F_1(x)-F_2(x))'=F'_1(x)-F'_2(x)=f(x)-f(x)=0.$$



      Remains to prove that the antiderivative of $0$ is a constant function.






      share|cite|improve this answer









      $endgroup$


















        1












        $begingroup$

        Hint:



        If $F_1,F_2$ are primitives of $f$, by linearity of differentiation,



        $$(F_1(x)-F_2(x))'=F'_1(x)-F'_2(x)=f(x)-f(x)=0.$$



        Remains to prove that the antiderivative of $0$ is a constant function.






        share|cite|improve this answer









        $endgroup$
















          1












          1








          1





          $begingroup$

          Hint:



          If $F_1,F_2$ are primitives of $f$, by linearity of differentiation,



          $$(F_1(x)-F_2(x))'=F'_1(x)-F'_2(x)=f(x)-f(x)=0.$$



          Remains to prove that the antiderivative of $0$ is a constant function.






          share|cite|improve this answer









          $endgroup$



          Hint:



          If $F_1,F_2$ are primitives of $f$, by linearity of differentiation,



          $$(F_1(x)-F_2(x))'=F'_1(x)-F'_2(x)=f(x)-f(x)=0.$$



          Remains to prove that the antiderivative of $0$ is a constant function.







          share|cite|improve this answer












          share|cite|improve this answer



          share|cite|improve this answer










          answered Dec 11 '18 at 11:02









          Yves DaoustYves Daoust

          128k675227




          128k675227























              0












              $begingroup$

              Let $G(x)= F_1 - F_2.$



              We observe that $G'(x)=0$. We claim this means that $G(x)$ is constant.



              Suppose on the contrary the function is non-constant this means there must exist $a, b in I$ and where $a<b$ such that $G(a) neq G(b)$.



              We consider $[a,b]$. Now We apply the mean value theorem to this interval and the function $G(x)$. We get that there must exist a $c$ such that:
              Which is a contradiction.
              $$G'(c)= frac{G(b)-G(a)}{b-a} land G'(c)=0,$$
              $$ frac{G(b)-G(a)}{b-a}=0 implies G(a)=G(b). $$
              We conclude that $G(x)$ is constant. Thus, $F_2$ and $F_1$ differ by a constant $square$.






              share|cite|improve this answer











              $endgroup$


















                0












                $begingroup$

                Let $G(x)= F_1 - F_2.$



                We observe that $G'(x)=0$. We claim this means that $G(x)$ is constant.



                Suppose on the contrary the function is non-constant this means there must exist $a, b in I$ and where $a<b$ such that $G(a) neq G(b)$.



                We consider $[a,b]$. Now We apply the mean value theorem to this interval and the function $G(x)$. We get that there must exist a $c$ such that:
                Which is a contradiction.
                $$G'(c)= frac{G(b)-G(a)}{b-a} land G'(c)=0,$$
                $$ frac{G(b)-G(a)}{b-a}=0 implies G(a)=G(b). $$
                We conclude that $G(x)$ is constant. Thus, $F_2$ and $F_1$ differ by a constant $square$.






                share|cite|improve this answer











                $endgroup$
















                  0












                  0








                  0





                  $begingroup$

                  Let $G(x)= F_1 - F_2.$



                  We observe that $G'(x)=0$. We claim this means that $G(x)$ is constant.



                  Suppose on the contrary the function is non-constant this means there must exist $a, b in I$ and where $a<b$ such that $G(a) neq G(b)$.



                  We consider $[a,b]$. Now We apply the mean value theorem to this interval and the function $G(x)$. We get that there must exist a $c$ such that:
                  Which is a contradiction.
                  $$G'(c)= frac{G(b)-G(a)}{b-a} land G'(c)=0,$$
                  $$ frac{G(b)-G(a)}{b-a}=0 implies G(a)=G(b). $$
                  We conclude that $G(x)$ is constant. Thus, $F_2$ and $F_1$ differ by a constant $square$.






                  share|cite|improve this answer











                  $endgroup$



                  Let $G(x)= F_1 - F_2.$



                  We observe that $G'(x)=0$. We claim this means that $G(x)$ is constant.



                  Suppose on the contrary the function is non-constant this means there must exist $a, b in I$ and where $a<b$ such that $G(a) neq G(b)$.



                  We consider $[a,b]$. Now We apply the mean value theorem to this interval and the function $G(x)$. We get that there must exist a $c$ such that:
                  Which is a contradiction.
                  $$G'(c)= frac{G(b)-G(a)}{b-a} land G'(c)=0,$$
                  $$ frac{G(b)-G(a)}{b-a}=0 implies G(a)=G(b). $$
                  We conclude that $G(x)$ is constant. Thus, $F_2$ and $F_1$ differ by a constant $square$.







                  share|cite|improve this answer














                  share|cite|improve this answer



                  share|cite|improve this answer








                  edited Dec 11 '18 at 12:04

























                  answered Dec 11 '18 at 11:58









                  Wesley StrikWesley Strik

                  2,057423




                  2,057423






























                      draft saved

                      draft discarded




















































                      Thanks for contributing an answer to Mathematics Stack Exchange!


                      • Please be sure to answer the question. Provide details and share your research!

                      But avoid



                      • Asking for help, clarification, or responding to other answers.

                      • Making statements based on opinion; back them up with references or personal experience.


                      Use MathJax to format equations. MathJax reference.


                      To learn more, see our tips on writing great answers.




                      draft saved


                      draft discarded














                      StackExchange.ready(
                      function () {
                      StackExchange.openid.initPostLogin('.new-post-login', 'https%3a%2f%2fmath.stackexchange.com%2fquestions%2f3035158%2fshow-that-two-primitives-antiderivatives-are-related-via-a-constant%23new-answer', 'question_page');
                      }
                      );

                      Post as a guest















                      Required, but never shown





















































                      Required, but never shown














                      Required, but never shown












                      Required, but never shown







                      Required, but never shown

































                      Required, but never shown














                      Required, but never shown












                      Required, but never shown







                      Required, but never shown







                      Popular posts from this blog

                      Plaza Victoria

                      In PowerPoint, is there a keyboard shortcut for bulleted / numbered list?

                      How to put 3 figures in Latex with 2 figures side by side and 1 below these side by side images but in...